D and C
D seems like the logical conclusion and everything else is a premise. Why couldn't this work:...
tomgbean on December 4, 2019
  • June 2016 LSAT
  • SEC3
  • Q18
2
Replies
Why E?
I thought this was a percent flaw.
ca_teran1@yahoo.com on November 12, 2019
  • June 2016 LSAT
  • SEC3
  • Q5
1
Reply
Could You Explain
This is probably one of the easiest MP questions ever but I'm not seeing how the correct answer c...
teddyteddted on November 12, 2019
  • June 2016 LSAT
  • SEC3
  • Q18
2
Replies
Why A?
Can someone please explain why the answer is A?
lerondagates on October 25, 2019
  • June 2016 LSAT
  • SEC3
  • Q8
1
Reply
Why E?
Could you please explain why E is the best answer? It seems like quite a logical leap to me.
hannahnaylor5 on October 24, 2019
  • June 2016 LSAT
  • SEC3
  • Q7
1
Reply
Could someone explain the correct answer choice?
I was stuck between A and B on this one and am having trouble following the argument through to t...
Hannah-Anderson on October 7, 2019
  • June 2016 LSAT
  • SEC3
  • Q6
1
Reply
Help
This question was very confusing for me. Can someone please explain? Thanks!
Minerva on September 5, 2019
  • June 2016 LSAT
  • SEC3
  • Q20
1
Reply
Please explain
why B is correct answer and not A?
amiru77 on May 8, 2019
  • June 2016 LSAT
  • SEC3
  • Q16
3
Replies
How?
I do not understand the answer to this question
crichburg1 on January 26, 2019
  • June 2016 LSAT
  • SEC3
  • Q19
3
Replies
Help please
Can someone walk me through this I'm completely lost.
alymathieu on January 22, 2019
  • June 2016 LSAT
  • SEC3
  • Q24
1
Reply
Confused
I was wondering why the flaw for this question would be an unrepresented sample flaw rather than ...
nb101 on October 12, 2018
  • June 2016 LSAT
  • SEC3
  • Q1
1
Reply
(D) and (E)
Curious about question (D) - is this not the right answer because it discusses "influences", wher...
hannah93092 on May 26, 2018
  • June 2016 LSAT
  • SEC3
  • Q24
3
Replies
Subsidiary Conclusion
How is the claim that there is a warm sea under the ice a subsidiary conclusion? I can see how it...
Parker-Zopp on May 22, 2018
  • June 2016 LSAT
  • SEC3
  • Q17
1
Reply
Explanation
Could I get some help with the explanation for this problem? I'm trying to understand how the spe...
Henleys on May 16, 2018
  • June 2016 LSAT
  • SEC3
  • Q22
2
Replies
Please Explain
Could you please explain the answer choice A?
rmkrutz@crimson.ua.edu on May 12, 2018
  • June 2016 LSAT
  • SEC3
  • Q21
1
Reply